Search found 170 matches


gmatmachoman wrote: seer bhai!
In my screen shot its showing
m> t ^(-4)
> 1/(t^4)
m>0

even if t is a imaginary number, m>0

SO pick D
yes, you are right. Thanks for pointing that gmatmachoman.

the explanation holds as long as t has an even power.

by iamseer

Tue May 11, 2010 8:25 am
Forum: Data Sufficiency
Topic: Inequalities
Replies: 4
Views: 1169

govind_raj_76 wrote: Sameer - How did you arrive at the one below ??

(16+32+64+128)/(1+2+4+8) =16
time taken is multiple of 30 for every subsequent distance r/2

first r/2 in 30 seconds
seconds r/2 in 60 seconds
and so on

and one round = 4*(r/2)

HTH!!

by iamseer

Mon May 10, 2010 11:14 pm
Forum: Problem Solving
Topic: sprinter starts running
Replies: 8
Views: 4511

the arguments speaks of all benefits (increase worker productivity, reduce absenteeism, and lessen insurance costs for employee health care) as far as the company is concerned and at the end jumps to conclude that these programs benefit both the company and the employees. If we have to strengthen th...

by iamseer

Mon May 10, 2010 12:38 pm
Forum: Critical Reasoning
Topic: 1000CR Question No 2 : Test 14
Replies: 4
Views: 2857

A majority of railway commuters reads or listens to music while traveling - is correct The majority of the students in this class are hard workers - is correct The majority of students are smart. A majority of students are smart. The majority of physics students are smart. A majority of physics stu...

by iamseer

Mon May 10, 2010 12:24 pm
Forum: Sentence Correction
Topic: The Majority Vs A majority!!
Replies: 2
Views: 2103

am sure experts will reply. In the mean while, here are my 2 cents. Collective noun takes a singular verb when the collection is thought of a one whole and it takes a plural verb when the individuals of which it is composed are thought of. The committee has issued its report The committee are divide...

by iamseer

Mon May 10, 2010 12:12 pm
Forum: Sentence Correction
Topic: The Majority Vs A majority!!
Replies: 2
Views: 2103

Most fruit is sprayed with dangerous pesticides before it is harvested, and is dangerous until it is washed . Clearly, the cafeteria is selling pesticide-covered fruit, If the fruit has been washed prior to reaching the cafeteria, is it dangerous for people to eat it? @Kevin: I am not arguing but r...

by iamseer

Sun May 09, 2010 7:25 pm
Forum: Critical Reasoning
Topic: The apples sold in this cafeteria are greasy
Replies: 37
Views: 7848

If B is false, could the conclusion still be valid? Yes: If the greasy residue is due to the fact that the apples have never been washed, any pesticide (greasy or not) would still be on the apples when they are sold. Thus B is NOT an assumption underying the argument. On the other hand, if A is fal...

by iamseer

Sun May 09, 2010 12:15 pm
Forum: Critical Reasoning
Topic: The apples sold in this cafeteria are greasy
Replies: 37
Views: 7848

Since y^2 is >0 we can cross multiply. from 1: x,y, Is (y^2*x^2-Y^3)>1? 2,3, Is (36-27)>1 Yes 0.1,0.2 (everything less than 1) NO Not Sufficient from 2: x^2,y^2, Is (y^2*x^2-Y^3)>1? take x=sqrt2 and y=sqrt3 2,3, Is (36-27)>1 Yes take x=0.1, y=0.2 0.01,0.04 (everything less than 1) NO Not Sufficient ...

by iamseer

Sun May 09, 2010 11:30 am
Forum: Data Sufficiency
Topic: Is x² - y > 1/y²
Replies: 1
Views: 1167

Didn't understand the explanation. As per the answer sheet, correct answer is A, although i am confused whether that is correct or not. for St 1: draw two line AD and BC on a piece of paper. Now draw a transversal CA such that angle y=40.... Now, with just this much information, one can plot the po...

by iamseer

Sun May 09, 2010 11:16 am
Forum: Data Sufficiency
Topic: Answer explanation needed.
Replies: 7
Views: 5938

vmt not equal 0. Therefore none of them is zero.
from 1:
m>v^2
therefore m >0
therefore v^2*m^3*t^4>0 (only m could make this negative, squares and forth powers are always positive)
Sufficient

from 2:
m>t^4
again m>0
therefore v^2*m^3*t^4>0
Sufficient

IMO answer D

by iamseer

Sun May 09, 2010 11:09 am
Forum: Data Sufficiency
Topic: Inequalities
Replies: 4
Views: 1169

from 1:
Point D could be anywhere on line AD
Insufficient

from 2:
Point A could be anywhere on line AD
Insufficient

Combining 1 and 2:
Sum of all angles of a triangle is 180.
Sufficient

IMO answer C

by iamseer

Sun May 09, 2010 11:00 am
Forum: Data Sufficiency
Topic: Answer explanation needed.
Replies: 7
Views: 5938

from 1:
x+2y>8
x,y,x+2y, Is x+y>6?
0,5,10, NO
4,4,12, YES
Not Sufficient

from 2:
Similarly like 1.
Not Sufficient

Combining 1 and 2:
3x+3y>16
x+y>16/3.... So it can be >6 or <6 or =6
Not sufficient

IMO answer E

by iamseer

Sun May 09, 2010 10:56 am
Forum: Data Sufficiency
Topic: Ds 1
Replies: 4
Views: 1251

from 1:
Since x>r and y<s, we are subtracting a smaller quantity from a bigger quantity. So, (x - y) > (r - s) is always true.
Sufficient.

from 2:
xy<rs

x,y,r,s,xy,rs, Is (x - y) > (r - s)
3,4,5,3,12,15, NO
3,-4,5,3,-12,15, YES

Not Sufficient

IMO answer A

by iamseer

Sun May 09, 2010 10:51 am
Forum: Data Sufficiency
Topic: DS2
Replies: 2
Views: 1222

IMO B

drop a perpendicular from B on AD.Let that point be X. BX has length of 3. AX =3 b'cos 45-45-90 triangle. Also, AB =3sqrt2
also, BC=8-3=5

therefore perimeter = 2*(5+3sqrt2) = 10 + 6sqrt2

HTH!!

by iamseer

Sun May 09, 2010 10:42 am
Forum: Problem Solving
Topic: Coordinate geometry
Replies: 5
Views: 1024

iamseer wrote:from 1:
y>0
therefore 3y >0
-2x>3y therefore -2x>0
therefore x has to be negative
Sufficient

from 2:
2x+5y=20
y=(20-2x)/5
3y= (60-6x)/5
-2x > (60-6x)/5
-10x > 60 - 6x
-4x>60

therefore x is negative.
Sufficient.

answer D

by iamseer

Sat May 08, 2010 11:05 pm
Forum: Data Sufficiency
Topic: DS-GMAT Prep
Replies: 4
Views: 1147